Difference between revisions of "2021 JMPSC Invitationals Problems/Problem 9"

Line 4: Line 4:
 
==Solution==
 
==Solution==
 
asdf
 
asdf
 +
 +
  
 
==See also==
 
==See also==
#[[2021 JMPSC Invitational Problems|Other 2021 JMPSC Invitational Problems]]
+
#[[2021 JMPSC Invitationals Problems|Other 2021 JMPSC Invitationals Problems]]
#[[2021 JMPSC Invitational Answer Key|2021 JMPSC Invitational Answer Key]]
+
#[[2021 JMPSC Invitationals Answer Key|2021 JMPSC Invitationals Answer Key]]
 
#[[JMPSC Problems and Solutions|All JMPSC Problems and Solutions]]
 
#[[JMPSC Problems and Solutions|All JMPSC Problems and Solutions]]
 
{{JMPSC Notice}}
 
{{JMPSC Notice}}

Revision as of 17:31, 11 July 2021

Problem

In $\triangle ABC$, let $D$ be on $\overline{AB}$ such that $AD=DC$. If $\angle ADC=2\angle ABC$, $AD=13$, and $BC=10$, find $AC.$

Solution

asdf


See also

  1. Other 2021 JMPSC Invitationals Problems
  2. 2021 JMPSC Invitationals Answer Key
  3. All JMPSC Problems and Solutions

The problems on this page are copyrighted by the Junior Mathematicians' Problem Solving Competition. JMPSC.png